Diferencia entre revisiones de «Compleja:ej-cap3.4»

De luz-wiki
Sin resumen de edición
Sin resumen de edición
 
(No se muestran 15 ediciones intermedias de 2 usuarios)
Línea 15: Línea 15:
<math>Res(f(z),e^{\frac{1}{4}i\pi})= \lim_{z \to e^\frac{1}{4}i\pi}\frac{(z-e^{\frac{1}{4}i\pi})+(z+1)}{4z^3}=\frac{1}{4}(e^{\frac{-1}{4}i\pi}+e^{\frac{-3}{4}i\pi})</math>
<math>Res(f(z),e^{\frac{1}{4}i\pi})= \lim_{z \to e^\frac{1}{4}i\pi}\frac{(z-e^{\frac{1}{4}i\pi})+(z+1)}{4z^3}=\frac{1}{4}(e^{\frac{-1}{4}i\pi}+e^{\frac{-3}{4}i\pi})</math>


<math>Res(f(z),e^{\frac{3}{4}i\pi})= \lim_{z \to e^\frac{3}{4}i\pi}(z-e^{\frac{3}{4}i\pi})\frac{z+1}{(z^4+1)}</math>


<math>Res(f(z),e^{\frac{3}{4}i\pi})= \lim_{z \to e^\frac{3}{4}i\pi}\frac{(z-e^{\frac{3}{4}i\pi})+(z+1)}{4z^3}=\frac{1}{4}(e^{\frac{-3}{2}i\pi}+e^{\frac{-9}{4}i\pi})</math>


Aplicando la fórmula de Euler <math>e^{i x} = \cos x + i\,\mbox{sen}\,x</math> llegamos al resultado de la integral real.
<math>{\displaystyle \int}_{c}f(z)=\int_{-\infty}^\infty\frac{x+1}{x^4+1}\,dx=2\pi i\frac{1}{4}
(e^{\frac{-1}{2}i\pi}+e^{\frac{-3}{4}i\pi}+e^{\frac{-3}{2}i\pi}+e^{\frac{-9}{4}i\pi})=\frac{\pi \sqrt{2}}{2}</math>
--[[Usuario:Oscar Rodriguez|Oscar Rodriguez]] 17:40 9 dic 2010 (UTC)


p.199
p.199
Línea 23: Línea 31:
----
----


3. Calcule <math>\int_0^{2\pi } \frac{1}{2-\text{Sin}(\theta )} \, d\theta </math> .
3. Calcule <math>\int_0^{2\pi } \frac{1}{2-\text{sin}(\theta )} \, d\theta </math> .
 
Primero haciendo la sustitución <math>\text{sin}(\theta )=\frac{1}{2i}\left(z-\frac{1}{z}\right)</math> y
 
<math>d\theta =\frac{dz}{iz}</math> . Dicha sustitución proviene del hecho que
<math>\sin \left[ \theta  \right]=\frac{{{e}^{i\theta }}-{{e}^{-i\theta }}}{2i}</math>,
<math>z={{e}^{i\theta }}\,</math> y  <math>dz=i{{e}^{i\theta }}d\theta\,</math>.


Primero haciendo la sustitución <math>\text{Sin}(\theta )=\frac{1}{2i z}\left(z-\frac{1}{z}\right)</math> y
Continuando con el problema, obtenemos la integral
<math>\mathrm  d\theta</math> es igual a <math>\frac {\mathrm  dz}{\mathrm i z}</math>
obtenemos la integral




<math>\int _c\frac{1}{2+\left(\frac{1}{2i z}\left(z-\frac{1}{z}\right) \right)}\frac{dz}{i z} </math>
<math>\int _c\frac{1}{2+\left(\frac{1}{2i z}\left(z-\frac{1}{z}\right) \right)}\frac{dz}{i z} </math>


Simplificando queda <math>\int _c\frac{2}{4i z -z^2+1}dz\text{  }\frac{(-1)}{(-1)}</math>     o
Simplificando queda <math>\int _c\frac{2}{4i z -z^2+1}dz\text{  }\frac{(-1)}{(-1)}</math>  


o      <math>\int _c\frac{-2}{ z^2-4i z-1}dz</math>


<math>\int _c\frac{-2}{ z^2-4i z-1}dz
Luego buscamos los polos del denomidador osea los polos de <math>{{z}^{2}}-4iz-1\,</math>
</math>
los cuales son <math> z=\overset{+}{-}\sqrt{3}i+2i\,</math>


Luego buscamos los polos del denomidador osea los polos de
solo tomamos el polo <math>-\sqrt{3}i+2i</math>
<math>z^2-4i z-1
</math>
   
los cuales son <math> z=\overset{+}{-}\sqrt{3}i+2i</math>


solo tomamos el polo <math>-Sqrt[3] i + 2 i</math>
porque es el único que esta dentro del circulo de radio uno, el cual es la región sobre la que estamos integrando.
 
porque es el único que esta dentro del circulo de radio 1, el cual es la región sobre la que estamos integrando.




Ahora obtenemos el Residuo de la funcion que es igual a
Ahora obtenemos el Residuo de la funcion que es igual a


<math>{\displaystyle lim_{z\longrightarrow(-\sqrt{3}i+2i )}}{\displaystyle \dfrac{z-(-\sqrt{3}i+2i )}{z^2 - 4 i z - 1}}=(-\frac{i}{2 \sqrt{3}})</math>
<math>\operatorname{Re}s(f(z),-\sqrt{3}i+2i)=\underset{z\to (-\sqrt{3}i+2i)}{\mathop{\lim }}\,\frac{z-(-\sqrt{3}i+2i)}{{{z}^{2}}-4iz-1}=\left( -\frac{i}{2\sqrt{3}i} \right)</math>


hay que recordar que sacamos un -2 de la integral por loque al multiplicar ese -2 por <math>-\frac{i}{2 \sqrt{3}}</math>
hay que recordar que sacamos un  
<math>-2\,</math> de la integral por lo que al multiplicar ese  
<math>-2\,</math>  por <math>-\frac{i}{2 \sqrt{3}}</math>


obtenemos <math>(2 I)/(2 Sqrt[3])</math>  
obtenemos <math>\frac{2i}{2 \sqrt{3}}</math>


y finalmente por la siguiente  definición
y finalmente por la definición


<math>\frac{1}{2\pi  i}\int _cf(z)dz=\left\{\text{Res}\left(f(z),z_1\right)+\text{...}+\text{Res}\left(f(z),z_k\right)\right\}
<math>\frac{1}{2\pi  i}\int _cf(z)dz=\left\{\text{Res}\left(f(z),z_1\right)+\text{...}+\text{Res}\left(f(z),z_k\right)\right\}
Línea 67: Línea 76:




4. Calcule <math>\int_0^{2\pi}\frac{1}{\left(1-2b\text{Cos}[\theta]+b^2\right)}\,d\theta </math> ,      <math>b>1</math>
4. Calcule <math>\int_0^{2\pi}\frac{1}{\left(1-2b\text{cos}[\theta]+b^2\right)}\,d\theta </math> ,      <math>b>1\,</math>


<math>\int_0^{2\pi}\frac{1}{\left(1-2b\text{Cos}[\theta]+b^2\right)}\,d\theta </math> ,      < math > b > 1 < /math >
<math>\int_0^{2\pi}\frac{1}{\left(1-2b\text{cos}[\theta]+b^2\right)}\,d\theta </math> ,      <math>b>1\,</math>
 
al sustituir  <math>\text{cos}[\theta]=\frac{1}{2}\left(z+\frac{1}{z}\right)</math>  
al sustituir  <math>\text{Cos}[\theta]=\frac{1}{2}\left(z+\frac{1}{z}\right)</math>  
    
    
y  <math>\mathrm  d\theta</math> es igual a <math>\frac {\mathrm dz}{\mathrm i z}</math>
y  <math>d\theta =\frac{dz}{iz}</math>. La sustitución proviene del hecho que
 
<math>\cos \left[ \theta  \right]=\frac{{{e}^{i\theta }}-{{e}^{-i\theta }}}{2}</math> y
 
<math>z={{e}^{i\theta }}\,</math> y <math>dz=i{{e}^{i\theta }}d\theta\,</math>.


obtenemos la integral   
Continuando con el problema, obtenemos la integral   


<math>\int _c\frac{1}{i(1-z b)(z-b)}dz</math>
<math>\int _c\frac{1}{i(1-z b)(z-b)}dz</math>
Línea 83: Línea 95:
obtenemos que los dos polos  que son  
obtenemos que los dos polos  que son  


<math>z=b\,</math>    y    <math>z=\frac{1}{b}</math>    como 


<math>z=b</math>    y    <math>z=\frac{1}{b}</math>    como 




<math>b>1</math>  
<math>b>1\,</math>  


entonces el unico polo que estaría en la región que
entonces el unico polo que estaría en la región que


nos interesa es  <math>z=1/b</math>
nos interesa es  <math>z=\frac{1}{b}</math>  


por lo tanto solo necesitamos calcular el residuo de   
por lo tanto solo necesitamos calcular el residuo de   


<math>f(z)</math>     para este polo.
 
<math>f(z)\,</math>   para este polo.
    
    


<math>{\displaystyle lim_{z\longrightarrow(\frac{1}{b})}}{\displaystyle \dfrac{z-(\frac{1}{b} )}{i(1-z b)(z-b)}}=(-\frac{i}{-1+b^2})</math>  
<math>\operatorname{Re}s(f(z),\tfrac{1}{b})=li{{m}_{z\longrightarrow \left( \frac{1}{b} \right)}}\frac{z-\left( \frac{1}{b} \right)}{i(1-zb)(z-b)}=\left( -\frac{i}{-1+{{b}^{2}}} \right)</math>




finalmente la integral


finalmete la integral
<math>\int_0^{2\pi}\frac{1}{\left(1-2b\text{cos}[\theta]+b^2\right)}\,d\theta</math>     
<math>\int_0^{2\pi}\frac{1}{\left(1-2b\text{Cos}[\theta]+b^2\right)}\,d\theta</math>     




Línea 112: Línea 125:




9. Pruebe que <math>2\int_{0}^{\pi}\frac{d\theta}{b+\cos\theta}=\int_{0}^{2\pi}\frac{d\theta}{b+\cos\theta},b>1</math>
<math>f(-\theta)=\frac{1}{b+\cos(-\theta)}</math> pero el coseno es una función
par por lo que <math>\cos\theta=\cos(-\theta)</math>
asi <math>f(-\theta)=f(\theta)</math> por lo que la función es par.
De los cursos de cálculo se sabe que:
Para integrales de funciones pares de -L a L con periodo 2L
<math>\int_{-L}^{L}f(\theta)d\theta=2\int_{0}^{^{L}}f(\theta)d\theta</math>
La integral <math>2\int_{0}^{\pi}\frac{d\theta}{b+\cos\theta}</math> calcula
el area bajo la curva sobre medio periodo, pero al multiplicar por
2, obtenemos el area total de un periodo entero.
La integral <math>\int_{0}^{2\pi}\frac{d\theta}{b+\cos\theta}</math> calcula
el area bajo la curva de un periodo completo.
Por lo tanto <math>2\int_{0}^{\pi}\frac{d\theta}{b+\cos\theta}=\int_{0}^{2\pi}\frac{d\theta}{b+\cos\theta}</math>


--[[Usuario:Carlos López Cobá|Carlos López Cobá]] 10:04 14 dic 2010 (UTC)





Revisión actual - 05:04 14 dic 2010

2.-Calcule

Tomamos la función compleja de la cual tomamos las raíces para determinar los polos.Los cuales estan dados por:

, , ,

por tanto,

para obtener los residuos aplicamos

Aplicando la regla de L´ Hopital obtenemos

Aplicando la fórmula de Euler llegamos al resultado de la integral real.

--Oscar Rodriguez 17:40 9 dic 2010 (UTC)

p.199

mfg-wiki 15:01 30 nov 2010 (UTC)


3. Calcule .

Primero haciendo la sustitución y

. Dicha sustitución proviene del hecho que , y .

Continuando con el problema, obtenemos la integral


Simplificando queda

o

Luego buscamos los polos del denomidador osea los polos de los cuales son

solo tomamos el polo

porque es el único que esta dentro del circulo de radio uno, el cual es la región sobre la que estamos integrando.


Ahora obtenemos el Residuo de la funcion que es igual a

hay que recordar que sacamos un de la integral por lo que al multiplicar ese por

obtenemos

y finalmente por la definición

la integral es igual a


4. Calcule ,

, al sustituir

y . La sustitución proviene del hecho que

y

y .

Continuando con el problema, obtenemos la integral

Buscasmos los polos de la función

obtenemos que los dos polos que son

y como


entonces el unico polo que estaría en la región que

nos interesa es

por lo tanto solo necesitamos calcular el residuo de


para este polo.



finalmente la integral



es igual a

--Pedro Pablo Ramírez Martínez 14:18 9 dic 2010 (UTC)


9. Pruebe que

pero el coseno es una función par por lo que

asi por lo que la función es par.

De los cursos de cálculo se sabe que:

Para integrales de funciones pares de -L a L con periodo 2L

La integral calcula el area bajo la curva sobre medio periodo, pero al multiplicar por 2, obtenemos el area total de un periodo entero.

La integral calcula el area bajo la curva de un periodo completo.

Por lo tanto

--Carlos López Cobá 10:04 14 dic 2010 (UTC)



Compleja:ej-cap1.1 Compleja:ej-cap1.2 Compleja:ej-cap1.3 Compleja:ej-cap1.4

Compleja:ej-cap2.1 Compleja:ej-cap2.2 Compleja:ej-cap2.3 Compleja:ej-cap2.4 Compleja:ej-cap2.5

Compleja:ej-cap3.1 Compleja:ej-cap3.2 Compleja:ej-cap3.3 Compleja:ej-cap3.4